[obm-l] Re: [obm-l] Re: [obm-l] Re: [obm-l] Re: [obm-l] Re: [obm-l] Re: [obm-l] Teoria dos números

2020-03-17 Por tôpico Pedro José
Boa noite!
Aí, como dizia minha falecida vó, são outros quinhentos.
Como nas propostas anteriores n era natural. Vamos seguir nessa linha, se
não for reformule o problema.
Seja f(n)=   n (427 - 90n - 70n^2 + 45n^3 + 18n^4)
f(0)=0 qualquer natural divide, portanto, é indiferente.
f(1)= 330
f(2)= 1230
É fácil verificar que mdc(330,1230)=30 então D<=30, onde D é o máximo
inteiro que divide f(n) para todo n natural.
f(n) = 7n +5n^4 + 8 n^5 mod 10.
f(0)=0 mod10
f(1)= 20 = 0 mod10
f(2)= 350= 0 mod10
f(3)= 2370 = 0 mod10
f(4)= 9500 = 0 mod10
f(5)= 28160 = 0 mod10
f(6)=68730 = 0 mod10
f(7)=146510 = 0 mod10
f(8)=282680 = 0 mod10
f(9)=505260 = 0 mod10
logo 10 | f(n) para qualquer n natural.

f(n) = n -n^3 mod 3
f(0) = 0 mod 3
f(1) = 0 mod 3
f(2)= -6 = 0 mod 3
logo 3| f(n) para todo n natural
então D = 30.

Saudações,
PJMS



Em ter., 17 de mar. de 2020 às 11:57, Israel Meireles Chrisostomo <
israelmchrisost...@gmail.com> escreveu:

> Sim é isso q eu quis dizer
>
> Em ter, 17 de mar de 2020 11:12, Carlos Gustavo Tamm de Araujo Moreira <
> g...@impa.br> escreveu:
>
>> Acho que a pergunta deve ser qual é o maior inteiro positivo que divide
>> essa expressão para todo valor de n ao mesmo tempo.
>>
>> On Tue, Mar 17, 2020 at 6:58 AM Pedro José  wrote:
>>
>>> Bom dia!
>>> Se você considerar a expressão n(427-90n-70n^2+45n^3+18n^4)
>>> D=|n(427-90n-70n^2+45n^3+18n^4)|
>>> Por exemplo, n=1
>>> D=330.
>>> Agora se liberar n para variar D tende a oo.
>>>
>>> Se n for raiz da expressão, também tende a oi, pois qualquer inteiro
>>> divide 0.
>>>
>>>
>>> Em seg, 16 de mar de 2020 22:16, Israel Meireles Chrisostomo <
>>> israelmchrisost...@gmail.com> escreveu:
>>>
 não entendi

 Em seg., 16 de mar. de 2020 às 22:01, Pedro José 
 escreveu:

> Para um dado n é o módulo do valor da expressão.
>
> Em seg, 16 de mar de 2020 21:49, Pedro José 
> escreveu:
>
>> Boa noite!
>> O módulo dessa expressão tende a oo. Não existe máximo.
>> Saudações,
>> PJMS
>>
>> Em seg, 16 de mar de 2020 20:36, Israel Meireles Chrisostomo <
>> israelmchrisost...@gmail.com> escreveu:
>>
>>> Qual é o maior inteiro que divide  n (427 - 90n - 70n^2 + 45n^3 +
>>> 18n^4)?
>>>
>>> --
>>> Israel Meireles Chrisostomo
>>>
>>> --
>>> Esta mensagem foi verificada pelo sistema de antivírus e
>>> acredita-se estar livre de perigo.
>>
>>
> --
> Esta mensagem foi verificada pelo sistema de antivírus e
> acredita-se estar livre de perigo.



 --
 Israel Meireles Chrisostomo

 --
 Esta mensagem foi verificada pelo sistema de antivírus e
 acredita-se estar livre de perigo.
>>>
>>>
>>> --
>>> Esta mensagem foi verificada pelo sistema de antivírus e
>>> acredita-se estar livre de perigo.
>>
>>
>> --
>> Esta mensagem foi verificada pelo sistema de antivírus e
>> acredita-se estar livre de perigo.
>
>
> --
> Esta mensagem foi verificada pelo sistema de antivírus e
> acredita-se estar livre de perigo.

-- 
Esta mensagem foi verificada pelo sistema de antiv�rus e
 acredita-se estar livre de perigo.



[obm-l] Re: [obm-l] Re: [obm-l] Re: [obm-l] Re: [obm-l] Re: [obm-l] Re: [obm-l] Teoria dos números

2019-05-04 Por tôpico Pedro José
Bom dia!
Obrigado!
Encontrei uma demonstração, mas não tive bagavem para enrender. Vou ler as
publicações.

Saudações,
PJMS

Em sáb, 4 de mai de 2019 11:57, Anderson Torres <
torres.anderson...@gmail.com escreveu:

> Em seg, 29 de abr de 2019 às 16:38, Pedro José 
> escreveu:
> >
> > Boa tarde!
> > Pelo menos consegui descobrir que se um inteiro z >= não puder ser
> escrito da forma z=4^k (8m+7), com m,k >=0 e m,k inteiros então ele pode
> ser representado por uma soma de três parcelas, todas quadrados perfeitos.
> > Já a demonstração, não consegui compreender.
> >
>
> Essa é a parte chata. Mas tem paper pra caramba!
>
> https://en.wikipedia.org/wiki/Legendre%27s_three-square_theorem
> Legendre's three-square theorem - Wikipedia
> https://www.sciencedirect.com/science/article/pii/0022314X74900249
> A new proof of the three squares theorem - ScienceDirect
> https://brilliant.org/wiki/fermats-sum-of-two-squares-theorem/
> Sum of Squares Theorems | Brilliant Math & Science Wiki
>
> https://mathoverflow.net/questions/223939/proving-legendres-sum-of-3-squares-theorem-via-geometry-of-numbers
> nt.number theory - Proving Legendre's Sum of 3 Squares Theorem via
> Geometry of Numbers - MathOverflow
> https://core.ac.uk/download/pdf/82306476.pdf
> PII: 0022-314X(74)90024-9 - 82306476.pdf
>
> https://www.ams.org/journals/proc/1957-008-02/S0002-9939-1957-0085275-8/S0002-9939-1957-0085275-8.pdf
> S0002-9939-1957-0085275-8.pdf
> http://pollack.uga.edu/finding3squares-6.pdf
> finding3squares-6.pdf
> https://arxiv.org/pdf/0812.0540.pdf
> () - 0812.0540.pdf
>
>
>
> > Saudações,
> > PJMS
> >
> > Em seg, 29 de abr de 2019 às 14:14,  escreveu:
> >>
> >>
> >> Em 29 de abr de 2019 11:37, Pedro José  escreveu:
> >>
> >> Bom dia!
> >>
> >> Gostei desse problema. Fiz um montão de exemplos com números que não
> podem ser escritos como 4^n(8n+7) e todos puderam ser escritos como a soma
> de três quadrados.
> >> Vale para todos? Se sim, alguém poderia indicar uma demonstração?
> >>
> >> Saudações,
> >> PJMS
> >>
> >> Em dom, 7 de abr de 2019 às 16:16, Pedro José 
> escreveu:
> >>
> >> Boa tarde!
> >> Fiquei na dúvida se algoritmo valia para demonstração. Mas salvo engano
> para demonstração de quais números aceitam raízes primitivas usa-se
> algoritmo.
> >> Mas, agora com mais calma, poderia ter usado indução.
> >> 1) Foi provado que não vale para n=0.
> >> 2) Supondo que não vale para n, não valeria para n+1, por absurdo.
> Pois, se valesse, teria que valer para n.
> >> Creio que teria ficado mais elegante.
> >>
> >> Saudações,
> >> PJMS
> >>
> >>
> >> Em dom, 7 de abr de 2019 às 07:41, matematica10complicada <
> profdouglaso.del...@gmail.com> escreveu:
> >>
> >> Obrigado irmão. Está correto sim.
> >> Douglas O.
> >>
> >> Em qui, 4 de abr de 2019 às 19:44, Pedro José 
> escreveu:
> >>
> >> Boa noite!
> >> Estou mal, mesmo. Ao invés de nenhum li qualquer. Tinha simulado dois,
> três, quatro e deram fora, já iria questionar.
> >> Mas vamos lá:
> >> 0^2 = 0 mod8; 1^2 = 1 mod8; 2^2 = 4 mod8 3^2= 1 mod8; 4^2 = 0 mod 8;
> 5^2 = 1 mod 8 6^2 = 4 mod 8 e 7^2 = 1 mod8;
> >> Portanto o quadrado de um número, ou dá 0 ou da 1 ou 4 na equivalência
> mod8.
> >>
> >> Caso n=0 ==> x=8k+7= 7 mod8. Como mod conserva a soma, não há como
> somar 3 parcelas do conjunto, mesmo com repetição, {0,1,4} e obter 7. Então
> n>0
> >>
> >> Para n>0
> >> x = 4^n*(8K+7) ==> x pertence a 2 |N seja x = a^2 + b^2 + c^2 com a, b,
> c pertencentes a |N - {0}. teríamos que ter a,b,c pares ou um deles par e
> dois ímpares.
> >> mas 4 | x ==> x= 0 mod4. Mas se w pertence a 2|N + 1 ==> w^2 = 1 mod4.
> e se y pertence a 2 |N ==> y^2 = 0 mod 4. Como temos dois ímpares e um par
> e como a soma se conserva temos que x = 2 mod4, absurdo. Portanto só sobra
> a, b, c pares Se a,b,c pares podemos escrevê-los como a= 2s; b=2t e c=2u
> com s,t,u naturais.
> >> x = a^2+b^2+c^2= 4(s^2+t^2+u^2) ==> x1 = 4^(n-1) * (8m+7) = s^2+t^2+u^2
> e vale o mesmo raciocínio de que s,t,u são pares e poderão ser escritos
> como s=2f; t=2g; u= 2h, com f, g, h naturais e seguir nesse algoritmo até
> que tenhamos xj=4^0(8m+7)= p^2+q^2+r^2, absurdo. Pois, já vimos que n= 0
> não atende.
> >>
> >> Espero estar correto.
> >>
> >> Saudações.
> >>
> >>
> >>
> >>
> >>
> >> Em qua, 3 de abr de 2019 às 15:36, matematica10complicada <
> profdouglaso.del...@gmail.com> escreveu:
> >>
> >> Mostre que nenhum número da forma (4^n)(8k+7) , com n e k naturais pode
> ser escrito como soma de 3 tres quadrados
> >>
> >> Douglas Oliveira
> >>
> >> --
> >> Esta mensagem foi verificada pelo sistema de antivírus e
> >> acredita-se estar livre de perigo.
> >>
> >>
> >> --
> >> Esta mensagem foi verificada pelo sistema de antivírus e
> >> acredita-se estar livre de perigo.
> >>
> >>
> >> --
> >> Esta mensagem foi verificada pelo sistema de antivírus e
> >> acredita-se estar livre de perigo.
> >>
> >>
> >> --
> >> Esta mensagem foi verificada pelo sistema de antiv�rus e
> >> acredita-se estar livre de perigo.
> >>
> >>
> >>
> >> 

[obm-l] Re: [obm-l] Re: [obm-l] Re: [obm-l] Re: [obm-l] Re: [obm-l] Re: [obm-l] Teoria dos números

2018-03-20 Por tôpico Claudio Buffara
Seu orgulho talvez seja justificado!

Como você descobriu que qualquer terno ordenado da forma ( x , y , -(x+y)/2
) é solução da equação "sem o 1"?
Isso não me parece nem um pouco óbvio.

Eu sei que, dados três inteiros, pelo menos dois devem ter a mesma
paridade, e que, como a equação é simétrica em x,y,z, podemos supor spdg
que x e y têm a mesma paridade.
Mas daí a termos z = -(x+y)/2 é um salto bastante longo.

Além disso, supor uma solução com  z = -(x+y)/2 + h  para a equação
original (com o 1) também me parece uma sacada brilhante, ainda que leve a
um "salseiro".

[]s,
Claudio.




2018-03-20 16:33 GMT-03:00 Pedro José :

> Boa tarde!
>
> Ralph,
> parabéns pela sua resolução.
> Já, eu, caminhei por caminhos bem mais tortuosos.
> Embora extremamente deselegante é uma solução.
>
> Se xo,yo,zo é uma solução, temos que pelo menos duas incógnitas têm a
> mesma paridade.
> Como o problema é simétrico, sem perda de generalidade, vamos supor que xo
> e yo tenham a mesma paridade.
> então podemos escrever zo = -(xo+yo)/2 + h, com h inteiro.
> Sabendo-se que : [xo,yo, -(xo+yo)/2] é solução da equação sem o "1".
>
> e substituindo na equação original:
>
> h^3 + 2(xo+yo)h^2+ (3/4 (x+y)^2 + xy) h - 1=0.
>
> Como os coeficientes são inteiros as únicas possibilidades de h inteiro
> são 1 e -1.
>
> h=1.
>
> Seja a= xo +yo
>
> 3a^2 + 8a + 4xoyo=0
>
> xoyo >0, temos que 8|a| > 3a^2 ==> |a| < 8/3 ==>|a|=2, portanto xo= -1 e
> yo=-1(não podem ser positivos). Temos z=-(xo+yo)/2+h=0. (-1,-1,2) e suas
> permutações são soluções.
>
> xo.yo = 0 temos a=0 ou a= -8/3 (não atende) ==> xo=yo=0, z= 1. (0,0,1) e
> suas permutações são soluções.
>
> xo.y0 <0
>
> 3a^2 + 8a + 4xoyo=0
>
> para ter solução a inteiro:Δ = (8 + 6i)^2  ==> 64 - 48 xoyo =  64 + 96 i
> + 36 i^2 ==> xoyo = - (2i + 3î^2/4), com i par.
>
> a= i e xoyo = -(2i+ 3i^2/4) então xo e yo são soluções da equação t^2 -it
> - (2i +3i^2/4) = 0. i =2k; t^2 -2kt-(4k+3k^2)=0
>
> Δ = 4k^2 +16k + 12k^2 = 16k(k+1), que nunca será um quadrado perfeito com
> k<>0. Então não há soluções inteiras. (k=0, recaí em xoyo=0)
>
> h=-1
>
> Seja a= xo+yo
>
> -3a^2 +8a -( 4xoyo - 8) = 0
>
> 4xoyo> 8 ==> 8a >3a^2; a <=2; absurdo não atende 4xoyo>8.
>
> 4xoyo-8=0
>
> temos que a=o, não há inteiros que xoyo=2 e xo+yo=0.
>
> 4xoyo - 8 < 0
>
> Δ = (8 + 6i)^2 ; 64 - 48xoyo + 32 = 64 + 96 i + 36 i^2;  xoyo = -2/3 + 2i
> + 3/4i^2, xoyo não pertence aos inteiros não há solução.
>
>
> Ficam apenas: (0,0,1) ; (0,1,0); (1,0,0) ; (-1,-1,2); (-1,2;-1); (2,-1,-1)
>
> Ralph,
>
> Fiz esse salseiro todo, ao invés de fatorar. E olha, que ontem estava
> orgulhoso de ter achado a solução.
>
>
> Saudações,
> PJMS
>
>
>
> Em 20 de março de 2018 12:10, Pedro José  escreveu:
>
>> É acabou me ajudando. Resolvi de uma outra forma, mais complicada, usando
>> a fórmula. Quando tiver um tempo eu posto.
>>
>>
>> Em 19 de mar de 2018 21:02, "Ralph Teixeira" 
>> escreveu:
>>
>>> Opa, opa, opa! Pedro, voce achou uma formula assim generica, z=-(x+y)/2,
>>> que resolve esta equacao? Beleza, excelente ideia, temos um caminho!
>>>
>>> Porque, se z=-(x+y)/2 eh SEMPRE solucao disso (independente de
>>> "inteiros" ou nao), quer dizer que essa coisa horrorosa, passando tudo para
>>> o outro lado, tem z+(x+y)/2 como fator, ou seja, x+y+2z como fator.
>>> Analogamente, vai ter 2x+y+z e x+2y+z tambem!
>>>
>>> Em suma, a gente pode voltar na primeira equacao com a sua ideia, jogar
>>> tudo para a esquerda (exceto pelo 1 chato que nao aparece na sua
>>> expressao), e fatorar. Vejamos... Acho que fica assim:
>>>
>>> 1/2*(2x+y+z)(x+2y+z)(x+y+2z)=1
>>> (2x+y+z)(x+2y+z)(x+y+2z)=2
>>>
>>> Confiram se eu nao errei contas... Mas agora ficou **bem** facil! :D
>>>
>>> Abraco, Ralph.
>>>
>>> 2018-03-19 14:33 GMT-03:00 Pedro José :
>>>
 Bom dia!

 Estou só conjecturando. Pois, não consegui nenhuma restrição.
 A única coisa que consegui, mas não me adiantou de nada, é que:
 x,y pares ou x,y ímpares e z = -(x+y)/2 é solução de

 *(x + y)(y + z)(z + x)/2 + (x + y + z)3 =  – xyz*
 Também, não consegui provar que é a única família de solução da
 equação acima para inteiros.

 Em 19 de março de 2018 14:14, Claudio Buffara <
 claudio.buff...@gmail.com> escreveu:

> Podem existir soluções não triviais envolvendo inteiros negativos.
>
> 2018-03-19 10:17 GMT-03:00 Pedro José :
>
>> Bom dia!
>>
>> Poderia postar a solução? Não consegui achar nenhuma restrição para
>> trabalhar num subconjunto  pequeno dos inteiros.
>> Creio que vá ser apenas a trivial (0,0,1) e suas permutações.
>>
>> grato,
>> PJMS
>>
>> Em 13 de março de 2018 20:19, Douglas Oliveira de Lima <
>> profdouglaso.del...@gmail.com> escreveu:
>>
>>> Essa achei legal e estou postando.
>>>
>>> *Resolva nos inteiros a seguinte equação:  (x + y)(y + z)(z + x)/2 +
>>> 

[obm-l] Re: [obm-l] Re: [obm-l] Re: [obm-l] Re: [obm-l] RE: [obm-l] Re: [obm-l] RE: [obm-l] TEORIA DOS NÚMEROS

2012-03-25 Por tôpico Marcelo Salhab Brogliato
Vamos lá:
333^555 + 555^333 = 111^555 * 3^555 + 111^333 * 5^333 = 111^333 * 111^222 *
3^555 + 111^333 * 5^333

--

Como 97 é primo, pelo pequeno teorema de fermat, temos que: x^96 == 1 (mod
97).

Como 111 == 15 (mod 96) e 111 == 14 (mod 97), temos que: 111^111 == 14^15
(mod 97).
Mas, 14^2 == 2 (mod 97), então: 14^15 == 2^7 * 14 (mod 97) == 46

Então: 111^111 == 46 (mod 97).
Assim, 111^222 == 46^2 == 79 (mod 97).
Por fim, 111^333 == 46^3 == 79*46 == 45 (mod 97).

--

Voltando, temos: 111^333 * 111^222 * 3^555 + 111^333 * 5^333 == 45 * 79 *
3^555 + 45 * 5^333 (mod 97).

--

Como 555 == 75 (mod 96), temos: 3^555 == 3^75 (mod 97)
3^5 = 243 == 49 (mod 97), então: 3^75 == 49^15 (mod 97)
49^2 == 73 (mod 97), então: 49^15 == 73^7 * 49 (mod 97)
73^2 == 91 (mod 97), então: 73^7 * 49 == 91^3 * 73 * 49 == 70 (mod 97)
Enfim, 3^555 == 70 (mod 97)

Como 333 == 45 (mod 96), temos 5^333 == 5^45 (mod 97)
5^4 = 625 == 43 (mod 97), então: 5^45 == 43^11 * 5 (mod 97)
43^2 == 6 (mod 97), então: 43^11 * 5 == 6^5 * 43 * 5 = 36*36*6*43*5 == 45
(mod 97)
Enfim, 5^333 == 45 (mod 97)

--

Novamente, voltando, temos: 45 * 79 * 70 + 45 * 45 (mod 97).
Agora ficou fácil, hehe. =]

Concluindo, ficamos com 333^555 + 555^333 == 33 (mod 97).

Abraços,
Salhab




2012/3/24 Vanderlei * vanderma...@gmail.com

 *é mesmo? Então o enunciado está errado? Essa questão está no material do
 Poliedro, caderno do ITA número 1.

 *
 Em 24 de março de 2012 18:59, Bernardo Freitas Paulo da Costa 
 bernardo...@gmail.com escreveu:

 2012/3/24 Vanderlei * vanderma...@gmail.com:
  Pois é caro João, eu também cheguei nesse seu resto de 45. Mas vamos
  continuar na luta, alguma saída deve ter! A sua ideia parece ser muito
 boa,
  uma vez que o primeiro resultado é verdadeiro!
 O Maple 10 acha que

 333^555 + 555^333 mod 97 = 33...

 --
 Bernardo Freitas Paulo da Costa

 =
 Instruções para entrar na lista, sair da lista e usar a lista em
 http://www.mat.puc-rio.br/~obmlistas/obm-l.html
 =





[obm-l] Re: [obm-l] Re: [obm-l] Re: [obm-l] Re: [obm-l] Re: [obm-l] RE: [obm-l] Re: [obm-l] RE: [obm-l] TEORIA DOS NÚMEROS

2012-03-25 Por tôpico Vanderlei *
*obrigado Marcelo! Então o enunciado está errado mesmo! 97 não divide a
soma!

*
2012/3/25 Marcelo Salhab Brogliato msbro...@gmail.com

  Vamos lá:
 333^555 + 555^333 = 111^555 * 3^555 + 111^333 * 5^333 = 111^333 * 111^222
 * 3^555 + 111^333 * 5^333

 --

 Como 97 é primo, pelo pequeno teorema de fermat, temos que: x^96 == 1 (mod
 97).

 Como 111 == 15 (mod 96) e 111 == 14 (mod 97), temos que: 111^111 == 14^15
 (mod 97).
 Mas, 14^2 == 2 (mod 97), então: 14^15 == 2^7 * 14 (mod 97) == 46

 Então: 111^111 == 46 (mod 97).
 Assim, 111^222 == 46^2 == 79 (mod 97).
 Por fim, 111^333 == 46^3 == 79*46 == 45 (mod 97).

 --

 Voltando, temos: 111^333 * 111^222 * 3^555 + 111^333 * 5^333 == 45 * 79 *
 3^555 + 45 * 5^333 (mod 97).

 --

 Como 555 == 75 (mod 96), temos: 3^555 == 3^75 (mod 97)
 3^5 = 243 == 49 (mod 97), então: 3^75 == 49^15 (mod 97)
 49^2 == 73 (mod 97), então: 49^15 == 73^7 * 49 (mod 97)
 73^2 == 91 (mod 97), então: 73^7 * 49 == 91^3 * 73 * 49 == 70 (mod 97)
 Enfim, 3^555 == 70 (mod 97)

 Como 333 == 45 (mod 96), temos 5^333 == 5^45 (mod 97)
 5^4 = 625 == 43 (mod 97), então: 5^45 == 43^11 * 5 (mod 97)
 43^2 == 6 (mod 97), então: 43^11 * 5 == 6^5 * 43 * 5 = 36*36*6*43*5 == 45
 (mod 97)
 Enfim, 5^333 == 45 (mod 97)

 --

 Novamente, voltando, temos: 45 * 79 * 70 + 45 * 45 (mod 97).
 Agora ficou fácil, hehe. =]

 Concluindo, ficamos com 333^555 + 555^333 == 33 (mod 97).

 Abraços,
 Salhab




 2012/3/24 Vanderlei * vanderma...@gmail.com

 *é mesmo? Então o enunciado está errado? Essa questão está no material
 do Poliedro, caderno do ITA número 1.

 *
 Em 24 de março de 2012 18:59, Bernardo Freitas Paulo da Costa 
 bernardo...@gmail.com escreveu:

 2012/3/24 Vanderlei * vanderma...@gmail.com:
  Pois é caro João, eu também cheguei nesse seu resto de 45. Mas vamos
  continuar na luta, alguma saída deve ter! A sua ideia parece ser muito
 boa,
  uma vez que o primeiro resultado é verdadeiro!
 O Maple 10 acha que

 333^555 + 555^333 mod 97 = 33...

 --
 Bernardo Freitas Paulo da Costa

 =
 Instruções para entrar na lista, sair da lista e usar a lista em
 http://www.mat.puc-rio.br/~obmlistas/obm-l.html
 =






[obm-l] Re: [obm-l] Re: [obm-l] Re: [obm-l] RE: [obm-l] Re: [obm-l] RE: [obm-l] TEORIA DOS NÚMEROS

2012-03-24 Por tôpico Vanderlei *
*é mesmo? Então o enunciado está errado? Essa questão está no material do
Poliedro, caderno do ITA número 1.

*
Em 24 de março de 2012 18:59, Bernardo Freitas Paulo da Costa 
bernardo...@gmail.com escreveu:

 2012/3/24 Vanderlei * vanderma...@gmail.com:
  Pois é caro João, eu também cheguei nesse seu resto de 45. Mas vamos
  continuar na luta, alguma saída deve ter! A sua ideia parece ser muito
 boa,
  uma vez que o primeiro resultado é verdadeiro!
 O Maple 10 acha que

 333^555 + 555^333 mod 97 = 33...

 --
 Bernardo Freitas Paulo da Costa

 =
 Instruções para entrar na lista, sair da lista e usar a lista em
 http://www.mat.puc-rio.br/~obmlistas/obm-l.html
 =



[obm-l] Re: [obm-l] RE: [obm-l] Re: [obm-l] Re : [obm-l] Re: [obm-l] RE: [obm-l] Teoria dos números (2 questões simples)

2009-08-24 Por tôpico luiz silva


Ola,
 
Tente isso..acho q funciona .
 
1) b^2+ab+1 = 0 mod (a^2+ab+1)
 
2) a^2+ab+1= 0 mod (a^2+ab+1)
 
Substiutua (2) em (1)
 
Abs
Felipe
 
--- Em sex, 21/8/09, marcone augusto araújo borges 
marconeborge...@hotmail.com escreveu:


De: marcone augusto araújo borges marconeborge...@hotmail.com
Assunto: [obm-l] RE: [obm-l] Re: [obm-l] Re: [obm-l] Re: [obm-l] RE: [obm-l] 
Teoria dos números (2 questões simples)
Para: obm-l@mat.puc-rio.br
Data: Sexta-feira, 21 de Agosto de 2009, 21:38




#yiv1877891977 #yiv1193512529 .hmmessage P
{
margin:0px;padding:0px;}
#yiv1877891977 #yiv1193512529 {
font-size:10pt;font-family:Verdana;}

Hugo esclareceu,obrigado.Mas o Diogo soicitou ajuda em outra questão: se 
a^2+ab+1 divide b^2+ab+1 então a=b.Alguém poderia ajudar?
 


Date: Fri, 21 Aug 2009 16:34:51 -0700
From: luizfelipec...@yahoo.com.br
Subject: [obm-l] Re: [obm-l] Re: [obm-l] Re: [obm-l] RE: [obm-l] Teoria dos 
números (2 questões simples)
To: obm-l@mat.puc-rio.br






Hugo,
 
Valeu!!
Abs
Felipe

--- Em sex, 21/8/09, Hugo Fernando Marques Fernandes hfernande...@gmail.com 
escreveu:


De: Hugo Fernando Marques Fernandes hfernande...@gmail.com
Assunto: [obm-l] Re: [obm-l] Re: [obm-l] RE: [obm-l] Teoria dos números (2 
questões simples)
Para: obm-l@mat.puc-rio.br
Data: Sexta-feira, 21 de Agosto de 2009, 16:14



Todos os números são da forma 3k+1, 3k ou 3k-1. Como 3k não é primo, k1, então 
todos os primos maiores que 3 são da forma 3k+1 ou 3k-1.
 
Abraços.
 
Hugo.


2009/8/21 luiz silva luizfelipec...@yahoo.com.br






Ola Marcone,
 
Pq vc trabalha com primos da forma 3k+1 ou 3k-1..creio que nem todos os primos 
podem ser representados nesta forma...O correto não seria 2k+1 ou 2k-1 ??Ou 
então, se quiser representar primos maiores que 3, não seria correto trabalhar 
com primos da forma 3+2k?
 
Abs
Felipe

--- Em qui, 20/8/09, marcone augusto araújo borges 
marconeborge...@hotmail.com escreveu:


De: marcone augusto araújo borges marconeborge...@hotmail.com
Assunto: [obm-l] RE: [obm-l] Teoria dos números (2 questões simples)
Para: obm-l@mat.puc-rio.br
Data: Quinta-feira, 20 de Agosto de 2009, 21:10 



Olá,Diogo.Um comentário singelo:o único primo múltiplo de 3 é o próprio 3.Se a 
é primo e diferente de 3, então a=3k+1 ou a=3k-1,dai a^2=3p+1 e a^2 +2=3q, o 
que é uma contradição(pois a^2+2 é primo).Portanto a=3.Se eu estiver 
errado,certamente alguem irá corrigir.Um abraço.
 


Date: Thu, 20 Aug 2009 15:15:39 -0700
From: diog...@yahoo.com.br
Subject: [obm-l] Teoria dos números (2 questões simples)
To: obm-l@mat.puc-rio.br




Teoria dos números (2 questões simples)?
1. Mostre que se (a) e (a² + 2) são ambos primos então a=3
2. Mostre que se (a² +ab +1) divide (b² +ab + 1) então a=b.


Se puder ajudar, agradeço.


Veja quais são os assuntos do momento no Yahoo! + Buscados: Top 10 - 
Celebridades - Música - Esportes


Novo Internet Explorer 8: mais rápido e muito mais seguro. Baixe agora, é 
grátis! 





Veja quais são os assuntos do momento no Yahoo! + Buscados: Top 10 - 
Celebridades - Música - Esportes



Veja quais são os assuntos do momento no Yahoo! + Buscados: Top 10 - 
Celebridades - Música - Esportes


Instale o novo Internet Explorer 8 versão especial para o MSN. Download aqui 


  

Veja quais são os assuntos do momento no Yahoo! +Buscados
http://br.maisbuscados.yahoo.com